Unbestimmtes Integral = 0, und jetzt?

Aufrufe: 667     Aktiv: 08.06.2022 um 15:52

0
Hallo alle zusammen, ich soll folgendes zeigen:



Dafür habe ich ganz klassisch das Riemann S. Integral ausgrechnet. Ich bin auf das Ergebnis gekommen: Wenn ich



1/2pi*   lim ln(x^2+1) und jeweils a und -a eingesetzt. 
            a->

Wenn ich aber den Grenzwert jeweils bereichne kommt sowohl für a= als auch für a=-
∞ heraus. Am Ende hätten "[∞-]" was 0 ist. Das Integral verschwindet und existiert doch somit nicht oder? Was bedeutet diese Null ?


EDIT vom 08.06.2022 um 13:50:

Mögliche Lösung:
Diese Frage melden
gefragt

Punkte: 33

 
Kommentar schreiben
1 Antwort
2
Moin,
du sollst das Integral \(\int_{-\infty}^{\infty}x\cdot d(\frac{1}{2}+\frac{1}{\pi}\arctan{x})\) berechnen. Also zunächst F(x) differenzieren. Dann erhältst du \(\frac{1}{\pi}\int_{-\infty}^{\infty}\frac{x}{x^2+1}dx\). Am Ende erhälts du dann ein Ergebnis (deins ist richtig).
Wichtig ist: \(\infty -\infty \neq0\), sowas darf man sich auf keinen Fall angewöhnen. Falls etwas in der Art rauskommt, existiert das Integral nicht. Was man allerdings machen kann, z.B. bei \(\int_{-\infty}^{\infty}xdx\), was ja auch nicht existiert, ist, dass man den Cauchyschen Hauptwert bildet. Der wäre dann tatsächlich 0.
LG
Diese Antwort melden
geantwortet

Student, Punkte: 3.82K

 

Was genau ist der Cauchysche Hauptwert, sowas hatten wir nämlich noch gar nicht im Studium.   ─   huhu123 07.06.2022 um 20:23

Könnte ich das Integral auch aufspalten mit -∞ bis 0 und dann 0 bis ∞ ?   ─   huhu123 07.06.2022 um 21:41

könntest du machen, würde aber nichts ändern   ─   fix 08.06.2022 um 11:45

Ich habe folgendes gemacht, wobei tatsächlich 0 herausgekommen ist (siehe oben Foto).
Der Ausdruck im Limes kürzt sich weg, sodass am Ende der lim von 0 bleibt, was 0 ist. Würde das gehen?

  ─   huhu123 08.06.2022 um 13:48

Du darfst nicht \(a\) und \(-a\) gleichzeitig gegen unendlich laufen lassen. Deine Idee mit aufteilen war gut, mache einmal Integral 0 bis a und -a bis 0 und lassen in beiden Integrale, dann getrennt gegen unendlich laufen   ─   mathejean 08.06.2022 um 14:42

Stimmt, ich habe nur die Antwort gelesen. Aber nochmal @huhu123 ganz wichtig: \(\int_{\infty}^\infty \cdots \not = \lim_{a\to \infty} \int_{-a}^a\cdots\), also ist zumindest dein letzter Satz falsch, weil wir das uneigentliche Integral nicht untersucht haben   ─   mathejean 08.06.2022 um 15:16

Bis auf den letzten Satz, ist meine Rechnung richtig?   ─   huhu123 08.06.2022 um 15:39

stimmt, das hab ich falsch geschrieben, mit \(-\infty\) und \(\infty\) würde das Integral ja auch nicht existieren   ─   fix 08.06.2022 um 15:46

@huhu123 ja, es kommt 0 raus, die Rechnung stimmt bis auf kleine Notationsfehler: einmal fehlt ein dx, du darfst den lim nicht mit in die Klammer ziehen, und du solltest genauer ausführen, wieso jetzt 0 rauskommt   ─   fix 08.06.2022 um 15:51

Vielen Dank an jeden Einzelnen für die Hilfe!   ─   huhu123 08.06.2022 um 15:52

Kommentar schreiben